--> TS ∼ DST n°2 Soit $(u_n)$ la suite définie par $u_0=0$ et pour tout entier $n$ : $u_{n+1}=\sqrt{u_n+2}$.
  1. Donner une valeur approchée à $10^{-3}$ de $u_5$.
  2. À l'aide de la calculatrice, en tappant : 0 + entrée + $\sqrt{\text{Rep}+2}$ + entrée $\times$ 5, on obtient :
    $u_5\simeq1,998$.
  3. Montrer par récurrence que pour tout entier $n$, $\,\,0\leq u_n \leq 2$.
Initialisation
Pour $n=0$ : $u_0=0$, donc on a bien : $0\leq u_0\leq 2$.
Hérédité
Supposons que pour un certain entier $n$ : $0\leq u_n \leq 2$.
Montrons alors que : $0\leq u_{n+1} \leq 2$.

D'après l'hypothèse de récurrence :
$0$ $\leq$ $u_n$ $\leq$ $2$
$2$ $\leq$ $u_n+2$ $\leq$ $4$ En ajoutant $2$ à chacun des membres de l'encadrement
$\sqrt{2}$ $\leq$ $\sqrt{u_n+2}$ $\leq$ $\sqrt{4}$ En appliquant la fonction racine carrée qui est croissante sur $[0;+\infty[$
$\sqrt{2}$ $\leq$ $u_{n+1}$ $\leq$ $2$
$0$ $\leq$ $u_{n+1}$ $\leq$ $2$ Car un nombre plus grand que $\sqrt{2}$ est plus grand que $0$.
Conclusion
D'après le principe de récurrence, pour tout entier $n$, $\,\,0\leq u_n \leq 2$.
On considère les trois nombres complexes ci-dessous.
$z_1=\dfrac{-2}{2+i\sqrt{3}}$

$z_2 = (1+i)^4$

$z_3 = \dfrac{(6+2i)^2}{2-3i}$
  1. Donner leur forme algébrique.
  2. $z_1$ $=$ $\dfrac{-2}{2+i\sqrt{3}}$
    $=$ $\dfrac{-2}{2+i\sqrt{3}}\times\dfrac{2-i\sqrt{3}}{2-i\sqrt{3}}$
    $=$ $\dfrac{-4+2i\sqrt{3}}{2^2+(\sqrt{3})^2}$
    $=$ $\dfrac{-4+2i\sqrt{3}}{7}$
    $=$ $-\dfrac{4}{7}+\dfrac{2\sqrt{3}}{7}i$.
    $z_2$ $=$ $(1+i)^4$
    $=$ $((1+i)^2)^2)$
    $=$ $\left( 1+2i-1 \right)^2$
    $=$ $(2i)^2$
    $=$ $-4$.
    $z_3$ $=$ $\dfrac{(6+2i)^2}{2-3i}$
    $=$ $\dfrac{36+24i-4}{2-3i}\times\dfrac{2+3i}{2+3i}$
    $=$ $\dfrac{(32+24i)(2+3i)}{4+9}$
    $=$ $\dfrac{-8+144i}{13}$
    $=$ $-\dfrac{8}{13}+\dfrac{144}{13}i$.
  3. Déterminer celui dont le module est maximal.
On détermine tout d'abord le module de chacun de ces nombres à partir de leur forme algébrique.

$|z_1|$ $=$ $\sqrt{\left(-\dfrac{4}{7}\right)^2+\left( \dfrac{2\sqrt{3}}{7} \right)^2}$ $=$ $\sqrt{\dfrac{16}{49}+\dfrac{12}{49}}$ $=$ $\sqrt{\dfrac{28}{49}}$ $=$ $\dfrac{\sqrt{28}}{7}$ $=$ $\dfrac{2\sqrt{7}}{7}$ $\simeq$ $0,756$.

$|z_2|$ $=$ $\sqrt{(-4)^2+0^2}$ $=$ $4$.

$|z_3|$ $=$ $\sqrt{\left( -\dfrac{8}{13} \right)^2+\left( \dfrac{144}{13} \right)^2}$ $=$ $\sqrt{\dfrac{64}{169}+\dfrac{20\,736}{169}}$ $=$ $\dfrac{\sqrt{20\,800}}{13}$ $=$ $\dfrac{40\sqrt{13}}{13}$ $\simeq$ $11,094$.

Ainsi, c'est le nombre $z_3$ qui possède le plus grand module.
Résoudre dans $\mathbb{C}$ les équations suivantes. On écrira les solutions sous forme algébrique.
$z^2+z=-7$

$4z^2+2z-5=0$

$(9-2i)z^2=(2+4i)z$
$z^2+z=-7$ $\Leftrightarrow$ $z^2+z+7=0$

Le discriminant de cette équation vaut : $\Delta = 1^2-4\times7$ $=$ $-27$.

Ainsi, elle possède deux solutions complexes conjuguées :

$z_1=\dfrac{-1-i\sqrt{27}}{2}$ $=$ $\dfrac{-1-3\sqrt{3}}{2}$ $=$ $-\dfrac{1}{2}-\dfrac{3\sqrt{3}}{2}i$.

$z_2 = -\dfrac{1}{2}+\dfrac{3\sqrt{3}}{2}i$
$4z^2+2z-5=0$

Le discriminant de cette équation vaut : $\Delta =4+4\times4\times5$ $=$ $84$.

Ainsi, elle possède deux solutions réelles :

$z_1 = \dfrac{-2-\sqrt{84}}{8}$ $=$ $\dfrac{-2-2\sqrt{21}}{8}$ $=$ $\dfrac{-1-\sqrt{21}}{4}$

$z_2 = \dfrac{-1+\sqrt{21}}{4}$.
$(9-2i)z^2$ $=$ $(2+4i)z$
$(9-2i)z^2-(2+4i)z$ $=$ $0$
$z( (9-2i)z-(2+4i) )$ $=$ $0$


D'après la règle du produit nul :

$z=0$ ou $(9-2i)z-(2+4i) $ $=$ $0$
$(9-2i)z$ $=$ $2+4i$
$z$ $=$ $\dfrac{2+4i}{9-2i}$
$z$ $=$ $\dfrac{2+4i}{9-2i}\times\dfrac{9+2i}{9+2i}$
$z$ $=$ $\dfrac{10+40i}{81+4}$
$z$ $=$ $\dfrac{10}{85}+\dfrac{40}{85}i$
$z$ $=$ $\dfrac{2}{17}+\dfrac{8}{17}i$


L'équation admet donc deux solutions : $0$ et $\dfrac{2}{17}+\dfrac{8}{17}i$.
Donner les limites des suites définies ci-dessous :
$u_n = \dfrac{3}{2+n^2}$

$v_n=\dfrac{4n^2+1}{n(2n+1)}$

$w_n=\dfrac{\cos(2n)}{n+1}$
$\displaystyle{\lim_{n\rightarrow+\infty}}n^2=+\infty$, donc $\displaystyle{\lim_{n\rightarrow+\infty}}2+n^2=+\infty$ et $\displaystyle{\lim_{n\rightarrow+\infty}}\dfrac{3}{2+n^2}=0$.

Ainsi : $\displaystyle{\lim_{n\rightarrow+\infty}}u_n=0$.
$\displaystyle{\lim_{n\rightarrow+\infty}}v_n$ $=$ $\displaystyle{\lim_{n\rightarrow+\infty}}\dfrac{4n^2+1}{2n^2+n}$ $=$ $\displaystyle{\lim_{n\rightarrow+\infty}}\dfrac{4n^2}{2n^2}$ $=$ $2$. Pour tout entier $n$ :
$-1$ $\leq$ $\cos(n)$ $\leq$ $1$
$-\dfrac{1}{n+1}$ $\leq$ $\dfrac{\cos(n)}{n+1}$ $\leq$ $\dfrac{1}{n+1}$ car $n+1>0$
$-\dfrac{1}{n+1}$ $\leq$ $w_n$ $\leq$ $\dfrac{1}{n+1}$


Or, $\displaystyle{\lim_{n\rightarrow+\infty}}-\dfrac{1}{n+1}$ $=$ $\displaystyle{\lim_{n\rightarrow+\infty}}\dfrac{1}{n+1}$ $=$ $0$.

Ainsi, d'après le théorème d'encadrement des limites : $\displaystyle{\lim_{n\rightarrow+\infty}}w_n$ $=$ $0$.